LSAT and Law School Admissions Forum

Get expert LSAT preparation and law school admissions advice from PowerScore Test Preparation.

User avatar
 Dave Killoran
PowerScore Staff
  • PowerScore Staff
  • Posts: 5853
  • Joined: Mar 25, 2011
|
#26924
Complete Question Explanation
(The complete setup for this game can be found here: lsat/viewtopic.php?t=11277)

The correct answer choice is (B)

Answer choices (A), (C), (D), and (E): Each of these answer choices Could be true, and therefore each is incorrect.

Answer choice (B): This is the correct answer choice. This answer cannot occur because when Tape 3 is all R and Tape 2 is all H (and Tape 1 is J and J/F), then at least one F must be on Tape 4, and hence Tapes 3 and 4 contain music that violates the RF not-block.

Get the most out of your LSAT Prep Plus subscription.

Analyze and track your performance with our Testing and Analytics Package.